Saltar al contenido

¿Se puede derivar la expresión de la fuerza de Lorentz a partir de las ecuaciones de Maxwell?

Nuestro grupo redactor ha estado horas investigando la resolución a tu duda, te dejamos la resolución por esto deseamos servirte de mucha apoyo.

Solución:

Las ecuaciones de Maxwell no contienen ninguna información sobre el efecto de los campos en las cargas. Uno puede imaginar un universo alternativo donde los campos eléctricos y magnéticos no crean fuerzas en ninguna carga, pero las ecuaciones de Maxwell aún se mantienen. ($ vecE $ y $ vecB $ serían inobservables y totalmente inútiles de calcular en este universo, ¡pero aún podría calcularlos!) Por lo tanto, no puede derivar la ley de fuerza de Lorentz solo a partir de las ecuaciones de Maxwell. Es una ley aparte.

Sin embargo…

  • Algunas personas cuentan una versión amplia de la “ley de Faraday” como parte de las “ecuaciones de Maxwell”. La versión amplia de la ley de Faraday es “EMF = derivada del flujo” (a diferencia de la versión estrecha $ nablatimesvec E = -parcial_t vec B $). EMF se define como la ganancia de energía de las cargas que viajan a través de un circuito, por lo que esta ley brinda información sobre las fuerzas en las cargas, y creo que puedes derivar la fuerza de Lorentz a partir de aquí. (En comparación, $ nablatimesvec E = -parcial_t vec B $ habla de campos eléctricos y magnéticos, pero no dice explícitamente cómo o si esos campos afectan las cargas).

  • Algunas personas consideran que la ley de fuerza de Lorentz es esencialmente la definición de campos eléctricos y magnéticos, en cuyo caso es parte de la base sobre la que se construyen las ecuaciones de Maxwell.

  • Si asume la parte de la fuerza eléctrica de la ley de fuerza de Lorentz ($ vec F = q vec E $), Y asume la relatividad especial, puede derivar la parte de la fuerza magnética ($ vec F = q vec v times vec B $) de las ecuaciones de Maxwell, porque una fuerza eléctrica en un marco es magnética en otros marcos. El reverso también es true: Si asume la fórmula de la fuerza magnética y asume la relatividad especial, entonces puede derivar la fórmula de la fuerza eléctrica.

  • Si asume las fórmulas para la energía y/o el momento de los campos electromagnéticos, entonces la conservación de la energía y/o el momento implica que los campos tienen que generar fuerzas en las cargas, y presumiblemente puede derivar la ley de fuerza de Lorentz exacta.

No he visto esto mencionado en las respuestas, así que pensé que al menos debería mencionarlo. Si toma la perspectiva de que las ecuaciones de Maxwell son las ecuaciones que describen un campo de calibre $ U (1) $, entonces el acoplamiento mínimo (que es, en cierto sentido, la única forma invariable de calibre de acoplar materia a un campo de calibre) asegura que cualquier campo de calibre cargado la partícula obedece la ley de fuerza de Lorentz, siendo la única libertad el valor $e$ de su carga. Entonces, mientras que las propias ecuaciones de Maxwell, sin algunos supuestos adicionales, pueden no implicar necesariamente la ley de fuerza de Lorentz, $U(1)$ invariancia de calibre lo hace implica la ley de fuerza de Lorentz. De hecho, si toma la invariancia de calibre $U(1)$ como el punto de partida fundamental, entonces implica ambas cosas Las ecuaciones de Maxwell y la ley de fuerza de Lorentz. Nuevamente, esta es una cuestión de perspectiva, por lo que no estoy en desacuerdo con las otras respuestas, pero creo que este es el punto de vista moderno.

Sí, la ley de fuerza de Lorentz se puede derivar de las ecuaciones de Maxwell (hasta una constante multiplicativa), con solo unas pocas suposiciones sobre lo que significa hablar de una teoría de campo.

Si partimos de las ecuaciones de Maxwell en el vacío, observamos que son invariantes de Lorentz. Por lo tanto, esperamos que cualquier ley de fuerza sea mejor que sea invariante de Lorentz. Si lo desea, puede agregar esto como una suposición explícita.

Aplicando el teorema de Noether para la simetría de traslación del tiempo, obtenemos una ley de conservación de energía para una energía cuya densidad es $u=(1/8pi)(textbfE^2+textbfB^2)$. El factor de $1/8pi$ es arbitrario y no está especificado por el teorema de Noether. También existe la no unicidad en el sentido de que puede agregar ciertos tipos de términos a esta expresión que involucran cosas como las segundas derivadas de los campos, pero no creo que esos términos tengan ningún efecto en el siguiente argumento, porque el argumento dependerá solo de la integral de $u$, no en su densidad local, y los términos agregados solo dan términos superficiales en la integral, y estos desaparecen. Esta ambigüedad se discute en las conferencias de Feynman, sección II-27-4.

Ahora agregue los términos fuente a las ecuaciones de Maxwell. Considere dos láminas de carga $pm Q$ en forma de capacitor de placas paralelas con un espacio lo suficientemente pequeño para que el campo interior sea casi uniforme. La energía $U=int u dV$ es finita y calculable a partir de la geometría. Si acercamos una hoja a la otra en $dx$, la energía en el campo eléctrico cambia en $dU$. La fuerza total entre las hojas es $F_total=dU/dx$, que también podemos calcular.

Ahora, cuando hablamos de una teoría de campo, asumimos que es local en algún sentido. Por esta razón, la fuerza que actúa sobre una pequeña porción de carga $q$ en nuestro capacitor solo puede depender del campo en ese punto, no del campo en otro lugar. Pero el campo no tiene variación transversal, así que dado $textbfF_total$, podemos inferir la contribución $textbfF$ de la fuerza que actúa sobre $q$. El campo en realidad es discontinuo en nuestro ejemplo, pero uno puede lidiar con ese problema, lo que produce un factor de 2. El resultado de este ejemplo es $textbfF=qtextbfE$, y el único movimiento posible La habitación es que podríamos haber elegido una constante de proporcionalidad diferente en nuestra definición de $u$. En otras palabras, podríamos haber cambiado el factor de conversión entre energías electromagnéticas y energías mecánicas, pero aquí no teníamos otra libertad. Podríamos haber elegido este factor de conversión tal que $textbfF$ se desvanecería de forma idéntica, pero entonces los campos electromagnéticos serían indetectables con dispositivos materiales, por lo que esta posibilidad no es muy interesante.

Una vez que se establece la parte eléctrica de la ley de fuerza de Lorentz, la ley de fuerza de Lorentz completa se deriva de la invariancia de Lorentz.

Si para ti ha resultado provechoso este post, nos gustaría que lo compartas con el resto juniors de esta forma contrubuyes a dar difusión a este contenido.

¡Haz clic para puntuar esta entrada!
(Votos: 0 Promedio: 0)



Utiliza Nuestro Buscador

Deja una respuesta

Tu dirección de correo electrónico no será publicada. Los campos obligatorios están marcados con *